How many ways are there to to climb n steps (recurrence relations)?

Click For Summary
The discussion focuses on the recurrence relations for climbing n steps, emphasizing that the number of ways to climb n steps is represented by s_n. It clarifies that if Tom climbs the first step, he has n-1 steps left, leading to the equation s_n = s_{n-1}. Additionally, if he climbs two steps initially, it results in s_n = s_{n-2}. The participants agree that the initial equations provided were incorrect and should be revised to accurately reflect the recurrence relation. The thread concludes with a question about marking the discussion as "solved."
Eclair_de_XII
Messages
1,082
Reaction score
91
Homework Statement
Tom wants to climb ##n## steps. He can either climb one step at a time, or two steps at a time.
(a) How many ways are there for him to climb ##n## steps if he initially takes one step?
(b) How many ways are there if he initially takes two steps?
(c) Use these two answers in order to derive a recurrence relation for ##s_n##, the number of ways to climb ##n## steps.
Relevant Equations
The answer to (c) as Google-searched: ##s_n=s_{n-1}+s_{n-2}##.
(a) Okay, so if Tom climbs the first step, then he has ##n-1## steps to climb. So the number of ways to climb ##n## steps given he has initially climbed one, is ##s_n=s_{n-1}##.
(b) Similarly, ##s_n=s_{n-2}##.
 
Physics news on Phys.org
The words you have written are correct but the equations are not. ##s_n## represents the number of ways to climb n steps, not the number of ways to climb n steps when the first step is 1, nor the number of ways to climb n steps when the first step is 2. So you need to delete the characters "##s_n=##" from your answers.

The recurrence relation is what you have already written under 'Relevant Equations'.
 
So I just add the two answers togther?
 
Err, how do you mark this thread as "solved"?
 
Question: A clock's minute hand has length 4 and its hour hand has length 3. What is the distance between the tips at the moment when it is increasing most rapidly?(Putnam Exam Question) Answer: Making assumption that both the hands moves at constant angular velocities, the answer is ## \sqrt{7} .## But don't you think this assumption is somewhat doubtful and wrong?

Similar threads

  • · Replies 11 ·
Replies
11
Views
2K
  • · Replies 5 ·
Replies
5
Views
1K
Replies
5
Views
2K
  • · Replies 4 ·
Replies
4
Views
3K
Replies
2
Views
1K
  • · Replies 6 ·
Replies
6
Views
2K
  • · Replies 1 ·
Replies
1
Views
1K
  • · Replies 4 ·
Replies
4
Views
1K
  • · Replies 2 ·
Replies
2
Views
2K
Replies
3
Views
2K